pinkster104
pinkster104
09-09-2014
Mathematics
contestada
What is -3/4+ 3 1/4 x 1 1/5
Respuesta :
mariamikayla
mariamikayla
09-09-2014
[tex]-\frac{3}{4}+3\frac{1}{4}*1\frac{1}{5}= -\frac{3}{4}+\frac{13}{4}*\frac{6}{5}= \\ \\ =-\frac{3}{4}^{(5}+\frac{39}{10}^{(2}= \\ \\ =\frac{-15+78}{20}= \\ \\ =\boxed{\frac{63}{20}}[/tex]
Answer Link
VER TODAS LAS RESPUESTAS ( 18+ )
Otras preguntas
Which of the following actions should a PCT take to help prevent health-care associated infections? a) Limit hand hygiene practices b) Encourage sick patients t
If 36 grams of tin 4 phosphate is mixed with an excess of sodium carbonate ,how many grams of tin 4 carbonate will form
Joe Bageant's family's annual deer hunt taught him about safety and respect?a) Trueb) False
A 26-year-old woman presents to the ED because of palpitations, dizziness, syncope, and fatigue. She is diagnosed with mural pregnancy and paroxysmal supraventr
What is the largest exponent in a single term in a polynomial? a) 3b) 5c) 9d) 10
Describe the potential benefits of improving HR processes or mitigating risk in your workplace. Which of the following options best describes this scenario?
Based on anticipated changes in the future, how can health care marketing strategies change? Consider external influences that may guide strategy changes.
Calculate the slope and y-intercept predicted according to the Nernst equation. The Nernst equation is given by E = E^(0) - (2.303RP)/(nFlogQ), where Q = ([Fe^(
Touching behavior that is employed in every aspect of dental health care delivery but particular at chair side is?
dx? daily chronic cough w/ thick, mucopurulent, foul-smelling sputum, hemoptysis, CT showing airway dilation, lack of tapering of bronchi, bronchial wall thicki